[Linear Algebra] Construct an n x 3 matrix D such that AD=I3

Click For Summary
SUMMARY

The discussion focuses on constructing an n x 3 matrix D such that AD = I3, where A is a 3 x n matrix with columns spanning R3. Theorem 4 establishes that if A has a pivot position in every row, then for each vector b in R3, the equation Ax = b has a solution. The solution for D can be derived using the formula D = A^T(AA^T)^{-1}, which minimizes the squared Frobenius norm. Additionally, the discussion highlights the importance of understanding Lagrange Multipliers and Singular Value Decomposition for deriving this result.

PREREQUISITES
  • Understanding of matrix multiplication and dimensions
  • Familiarity with linear combinations and span of vectors
  • Knowledge of Theorem 4 in linear algebra
  • Basic concepts of matrix inversion and norms
NEXT STEPS
  • Study the derivation of D = A^T(AA^T)^{-1} in detail
  • Learn about Lagrange Multipliers and their application in optimization problems
  • Explore Singular Value Decomposition (SVD) and its relevance in linear algebra
  • Practice problems involving the construction of matrices that satisfy specific equations
USEFUL FOR

Students and professionals in mathematics, particularly those studying linear algebra, as well as educators seeking to deepen their understanding of matrix theory and its applications.

bornofflame
Messages
56
Reaction score
3

Homework Statement


Suppose that A is a 3 x n matrix whose columns span R3. Explain how to construct an n x 3 matrix D such that AD = I3.

"Theorem 4"
For a matrix A of size m x n, the following statements are equivalent, that is either all true or all false:
a. For each b in Rm, Ax = b has a solution
b. Each b in Rm is a linear combination of A.
c. The columns of A span Rm.
d. A has a pivot position in every row.

Homework Equations



AD = I3
Ax = b
I3 = [e1 e2 e3]
AB = [Ab1 ... bn]

The Attempt at a Solution



So far what I've managed to put together is the following:
Since the columns of A span R3, it follows that, per Theorem 4:
For each b in R3, Ax = b has a solution
Each b in R3 is a linear combination of A.
A has a pivot position in every row.

Because we can write AB = [Ab1 ... bn], D as [d1 d2 d3] and I3 as [e1 e2 e3], we can also write
AD = [Ad1 Ad2 Ad3] = [e1 e2 e3].
We can substitute Ad1 = e1 for Ax = b in this case and we know that, since the columns of A span R3 that this equation has a solution. This logic can be applied to the columns [d1 d2 d3] to solve for D.

Is this correct? If not, am I at least on the right track? Thanks.
 
Physics news on Phys.org
bornofflame said:

Homework Statement


Suppose that A is a 3 x n matrix whose columns span R3. Explain how to construct an n x 3 matrix D such that AD = I3.

"Theorem 4"
For a matrix A of size m x n, the following statements are equivalent, that is either all true or all false:
a. For each b in Rm, Ax = b has a solution
b. Each b in Rm is a linear combination of A.
c. The columns of A span Rm.
d. A has a pivot position in every row.

Homework Equations



AD = I3
Ax = b
I3 = [e1 e2 e3]
AB = [Ab1 ... bn]

The Attempt at a Solution



So far what I've managed to put together is the following:
Since the columns of A span R3, it follows that, per Theorem 4:
For each b in R3, Ax = b has a solution
Each b in R3 is a linear combination of A.
A has a pivot position in every row.

Because we can write AB = [Ab1 ... bn], D as [d1 d2 d3] and I3 as [e1 e2 e3], we can also write
AD = [Ad1 Ad2 Ad3] = [e1 e2 e3].
We can substitute Ad1 = e1 for Ax = b in this case and we know that, since the columns of A span R3 that this equation has a solution. This logic can be applied to the columns [d1 d2 d3] to solve for D.

Is this correct? If not, am I at least on the right track? Thanks.

Looks fine to me.
 
  • Like
Likes bornofflame
I agree with the above.

extension:
Since you are in working in reals, you have an inner product, and may may want to consider pushing the problem a bit to find the solution with the minimum length --i.e. getting the minimum length (squared 2 norm) solution for each vector ##\mathbf d_1##, ##\mathbf d_2## and ##\mathbf d_3## (equivalently, minimum squared Frobenius norm for ##D##). Specifically said solution would be given by

##D = A^T\big(AA^T\big)^{-1}##
or
##\mathbf d_j = A^T\big(AA^T\big)^{-1} \mathbf e_j##
- - - -
There's a way to derive this via Lagrange Multipliers, and another way to derive it via Singular Value Decomposition. Both are instructive. This is a very nice result, that not a lot of people are familiar with, for some reason.
- - - -
it also has a nice common sense feel to it, because

##I_3 = AD = A\Big(D\Big) = A\Big( A^T\big(AA^T\big)^{-1}\Big) = A A^T\big(AA^T\big)^{-1}= \big(A A^T\big)\big(AA^T\big)^{-1} = I_3##

so to get comfortable with this solution, you'd just need to convince yourself that ##\big(AA^T\big)## is in fact non-singular.
 
Last edited:
  • Like
Likes bornofflame
Thank you, both. I'm still not comfortable with Linear Alebra yet and so feel quite shaky in what I've learned. Unfortunately there are no tutors for it on campus.

Thanks for the extension, StoneTemplePython. We haven't touched Lagrange Multipliers or Singular Value Decomposition yet, but that's something to look forward to.
 
Question: A clock's minute hand has length 4 and its hour hand has length 3. What is the distance between the tips at the moment when it is increasing most rapidly?(Putnam Exam Question) Answer: Making assumption that both the hands moves at constant angular velocities, the answer is ## \sqrt{7} .## But don't you think this assumption is somewhat doubtful and wrong?

Similar threads

Replies
15
Views
2K
  • · Replies 3 ·
Replies
3
Views
3K
  • · Replies 7 ·
Replies
7
Views
2K
  • · Replies 3 ·
Replies
3
Views
1K
  • · Replies 8 ·
Replies
8
Views
3K
Replies
5
Views
2K
  • · Replies 2 ·
Replies
2
Views
3K
  • · Replies 5 ·
Replies
5
Views
3K
  • · Replies 2 ·
Replies
2
Views
2K
  • · Replies 4 ·
Replies
4
Views
2K